Đến nội dung

supermember

supermember

Đăng ký: 01-09-2006
Offline Đăng nhập: 19-03-2024 - 20:53
****-

#743527 Bài 2 - Cuộc thi giải toán "Mừng xuân Giáp Thìn, mừng VMF tròn 20 tuổi"

Gửi bởi supermember trong 13-02-2024 - 22:53

Đáp án chính thức:

Xét $ A = \frac{ 5ab(a^2 +b^2 - 2)}{ 5ab-1} = \frac{ a^2 (5ab-1) +b^2 (5ab-1) + a^2 +b^2 - 2ab}{ 5ab-1} =  a^2 + b^2 + \frac{(a-b)^2}{ 5ab-1}$

Do đó : $  A = \frac{ 5ab(a^2 +b^2 - 2)}{ 5ab-1}$ là số nguyên dương khi và chỉ khi $\frac{(a-b)^2}{ 5ab-1}$ là số nguyên không âm $(*)$

 

Xét phương trình : $ \frac{(a-b)^2}{ 5ab-1} =k$ với $k$ là  $1$ hằng  số nguyên không âm $(1)$

 

Do vai trò $a \ ; \ b$ là như nhau nên từ nay, không mất tính tổng quát, ta chỉ cần xét đến trường hợp $ a \geq b$

 

$ (1)$ tương đương với: $ a^2 - (5bk + 2b) a + b^2 +k =0$ $(2)$

 

Coi $(2)$ là phương trình bậc $2$ ẩn $a$, $b$ là tham số.

 

Theo giải thiết bài toán, thì do tồn tại cặp số $(a; b)$ thỏa $(2)$ nên theo nguyên lý cực hạn, ta có thể chọn ra bộ số $(a_0 ; b_0)$  thỏa $(2)$ sao cho tổng $ a_0 + b_0 $ đạt giá trị nhỏ nhất có thể. $(3)$

 

Theo định lý Viette thì ngoài nghiệm $a_0$, $(2)$ sẽ còn có nghiệm $a_1$ thỏa mãn: $\left\{\begin{matrix} a_1 = 5b_0 k +2b_0 -a_0 \\ a_1 = \frac{b_0^{2}+k}{a_0} \end{matrix}\right.$

 

Mà từ đây ta dễ thấy $a_1$ cũng phải là số nguyên dương.

Theo đó, dễ thấy  cặp số nguyên dương $(a_1; b_0)$ cũng thỏa $(2)$ và từ cách chọn ra bộ số $(a_0 ; b_0)$ thì hiển nhiên:

$ a_1 + b_0 \geq a_0 + b_0 $

$ \implies  a_1 \geq a_0  \implies  \frac{b_0^{2}+k}{a_0} \geq a_0  \implies  k \geq a^{2}_0 -  b^{2}_0$
 
$\implies  \frac{(a_0 -b_0)^2}{ 5a_0 b_0-1} \geq a^{2}_0 -  b^{2}_0$ $(4)$

Đến đây, ta thấy là không thể xảy ra trường hợp $a_0 > b_0$ vì nếu $  a_0 > b_0$ thì từ $(4)$ suy ra:

$  \frac{a_0 -b_0}{ 5a_0 b_0-1} \geq a_0 +  b_0 \implies a_0 -b_0 \geq (a_0 +  b_0) ( 5a_0 b_0-1)$

Điều này không thể xảy ra do $ (a_0 +  b_0) ( 5a_0 b_0-1) \geq 4 (a_0 +  b_0) > a_0 - b_0$

 

Do đó, chỉ có thể xảy ra trường hợp $a_0 = b_0$. Suy ra:  $k=0$.

Tức là $ k= \frac{(a-b)^2}{ 5ab-1}$ là số nguyên không âm khi và chỉ khi $ a=b$ $(**)$

 

Từ $ (*); (**)$, ta kết luận:

 

$  A = \frac{ 5ab(a^2 +b^2 - 2)}{ 5ab-1}$ là số nguyên dương khi và chỉ khi $a=b$ (đpcm)
 
 

Chấm điểm:

1. trantiennguyen: Bài làm ý tưởng chứng minh đúng, mạch lạc, ngắn gọn. Tuy nhiên chữ viết quá xấu, vừa mờ, vừa khó đọc, đạt $9$ điểm.
2. habcy12345: Bài giải đầu tiên nộp bị sai nên tất nhiên không xét đến bài giải bổ sung, đạt $0$ điểm

3. mathproo: Bài giải bị sai ngay đoạn $\Rightarrow k=\frac{5b_{0}^2+m-2}{5a_{0}}>a_{0}>b_{0}$, đạt $0$ điểm.
4. Nguyen Bao Khánh: Bài giải tốt, sạch, đẹp, đạt $ 10$ điểm
5: bahieupbc: Lời giải tốt, điểm trừ Latex sai dẫn đến vài đoạn giám khảo khó đọc, đạt $9.5$ điểm

6: qminhdls: Lời giải viết không tốt, lúc thì $H$, lúc thì $T$ lung tung lẫn lộn. Ngoài ra cũng không chỉ ra rằng vai trò của $A;B$ như nhau nên chỉ cần xét $ A \geq B$. 

Nếu không có giải thiết 
$ A \geq B$ thì bất đẳng thức : $ \frac{ (A-B)^2}{ 5AB -1} \geq A- B$ là đúng chứ không sai.
Đạt $4$ điểm

7 huytran08: Lời giải sai, $0$ điểm
8 duc3290: lời giải ý tưởng đúng tuy nhiên ký hiệu $ \rightarrow$ là sai, chỉ có ký hiệu $ \implies$ hoặc $ \Leftrightarrow$ mà thôi, $8$ điểm . Lỗi trình bày này quá cơ bản và không đáng mắc phải.

Các thành viên nhận giải : Nguyen Bao Khanh, Tran Tien Nguyen, baohieupbc vui lòng xác minh thông tin (thẻ học sinh, sổ liên lạc, giấy khai sinh...) với giảm khảo hxthanh để chứng minh bản thân là học sinh THCS để nhận giải.

Các bạn lưu ý:

Có $2$ cách trình bày:
 

cách $1$: chứng minh không thể xảy ra trường hợp $ a > b$ , cũng không thể xảy ra trường hợp $ a<b$ , sau đó kết luận $a =b$ (như trantiennguyen trình bày)

Cách $2$: nhận xét đầy đủ: " Vai trò của $a; b $ là như nhau nên không mất tính tổng quát, ta chỉ cần xét trường hợp: $ a \geq b$ " (như Nguyen Bao Khanh trình bày) , sau đó đi chứng minh không thể xảy ra $ a >b$ rồi kết luận chỉ có thể xảy ra trường hợp $a =b$.

Nếu không nêu rõ vai trò $a; b $ như nhau mà giả sử ngay $ a \geq b$ thì sẽ không được tối đa điểm.




#743346 Tính $lim(x_n.y_n)$

Gửi bởi supermember trong 03-02-2024 - 19:27

Bài này đơn giản. Mấu chốt bài này là từ " cấu trúc lặp " :  $ x_{n+1} = (x^{2}_n - 2)^2 -2$ ta đoán ra ngay là có liên quan đến việc chuyển $x_1$ về dạng $ a + \frac{1}{a}$.

Thật vậy, Xét phương trình $ \alpha + \frac{1}{\alpha} = 3$, ta lưu ý đến nghiệm của phương trình này trên tập hợp $(2; + \infty)$

 

Ta thấy phương trình này chỉ có nghiệm duy nhất thuộc tập hợp $(2; + \infty)$ là : $ \alpha_1 = \frac{3 + \sqrt{5}}{2} $

 

Bằng quy nạp, ta dễ dàng chứng minh được:

$ x_n = \alpha^{4^{n-1}}_1 + \frac{1}{ \alpha^{4^{n-1}}_1}$ với mọi $n \geq 1$

Ngoài ra: $ y_n =  \frac{1}{ \alpha^{4^{n-1}}_1}$ với mọi $n \geq 1$

Suy ra : $ \lim_{n \to + \infty} (x_n \cdot y_n ) = \lim_{n \to + \infty}   \left( \alpha^{4^{n-1}}_1 + \frac{1}{ \alpha^{4^{n-1}}_1}  \right) \cdot  \frac{1}{ \alpha^{4^{n-1}}_1}$

$ = \lim_{n \to + \infty} \left( 1 +  \frac{1}{ \alpha^{2 \cdot 4^{n-1}}_1} \right) = 1$ ,

Ở đây do $ \alpha_1 >2 >1 $ nên hiển nhiên :  $\lim_{n \to + \infty} \frac{1}{ \alpha^{2 \cdot 4^{n-1}}_1} = 0$

Lâu ngày quá không giải Toán, hi vọng là không bị sai ở đâu đó :D

 

Nhận xét:

 

Bài này sẽ không chỉ dừng lại ở đây đâu, nếu tiếp tục mở rộng ra thêm nữa, ta có thể xây dựng các bài toán khó hơn nhiều cũng dựa vào cấu trúc: $  \alpha \pm \frac{1}{\alpha} $. Chẳng hạn như có thể xây dựng thành các dạng lũy thừa kiểu:  $\alpha^{3^n} \pm  \frac{1}{ \alpha^{3^n}}$ với mọi $n \geq 1$.

Cái này thì chắc trông chờ vào tài năng sáng chế bài toán của Hoàng Xuân Thanh để có các bài Toán hay cho mọi người cùng giải.




#742066 $ x_{1}=4$ và $x_{n+1}=45.x_{n}+...

Gửi bởi supermember trong 08-11-2023 - 18:44

Bài này mấu chốt là cần nhìn ra: $ 45^2 = 2025$

 

Ta có:  $ x_{n+1} - 45 x_n = \sqrt{ 2024 x^{2}_{n} +16}  \implies ( x_{n+1} - 45 x_n )^2 =  2024 x^{2}_{n} +16  \implies x^{2}_{n+1} - 90 x_{n+1} x_n   + 2025 x^{2}_{n} =  2024 x^{2}_{n} +16 $

 

$\implies x^{2}_{n+1} - 90 x_{n+1} x_n   +  x^{2}_{n} = 16 \  (*)$ 

 

tức là giờ ta sẽ tập trung tìm CTTQ của dãy $(x_n)_{n \geq 1}$ dựa vào đẳng thức $(*)$

 

Thật vậy, dễ thấy từ $(*)$ thì ta cũng có:

 

 

$\implies x^{2}_{n+2} - 90 x_{n+2} x_{n+1}   +  x^{2}_{n+1} = 16 \  (**)$

 

Lấy $2$ đẳng thức $(*)$ và $(**)$ trừ nhau vế theo vế, ta có:

 

 

$ x^{2}_{n+2} -  x^{2}_{n} -  90 x_{n+1}  ( x_{n+2}  - x_n)= 0$

 

$ ( x_{n+2} -  x_{n}) ( x_{n+2} +  x_{n} )  -  90 x_{n+1}  ( x_{n+2}  - x_n)= 0$

Suy ra $  ( x_{n+2} -  x_{n}) ( x_{n+2} +  x_{n}   -  90 x_{n+1} )= 0 \ (***)$

 

Mà rõ ràng từ giả thiết bài toán, ta thấy rõ ràng $ x_{n+1} > 45 x_n > x_n \implies x_{n+2} > x_n $ với mọi số nguyên dương $n$

 

Nên từ đẳng thức $(***)$ thì rõ ràng  $x_{n+2}    -  90 x_{n+1}  +  x_{n} = 0  \ \forall n \in \mathbb{N}^{*}$

Theo đó, dãy $(x_n)_{n \geq 1}$ là dãy truy hồi tuyến tính cấp $2$, phương trình đặc trưng của nó : $ y^2 -90y +1 =0$ có $2$ nghiệm 

$ y_{1; 2} = 45 \pm 2\sqrt{506}$ và dãy này theo đó sẽ có CTTQ dạng : $ x_n = A \cdot y^{n}_1 + B \cdot y^{n}_2 \ \forall n \in \mathbb{N}^{*}$

 

Trong đó, các hệ số $ A; B $ này được xác định thông qua việc giải hệ phương trình:

 

$\left\{\begin{matrix} x_1 = A y_1 + B y_2 & \\ x_2 = A y^2_{1} + By^2_{2} \end{matrix}\right.$

 

Trong đó, chú ý là theo định lý Viette thì tích $ y_1 \cdot y_2 = 1$ , tức là ta có thể viết hệ này dưới dạng:

 

 

$\left\{\begin{matrix} 4 = A y_1 + \frac{B}{y_1}  & \\ 360 = A y^2_{1} + \frac{B}{y^2_{1}} \end{matrix}\right.$

 

Thực ra cái này giờ đơn giản rồi, tính nhẹ nhàng thì ra được:

 

$ A = \frac{360y_1 - 4}{y^{3}_1 - y_1} ; B = 4y_1 - A \cdot y^{2}_1$

 

Tức là: $ x_n =  \frac{360y_1 - 4}{y^{3}_1 - y_1} \cdot y^{n}_{1} +  \left( 4y_1 - \left(  \frac{360y_1 - 4}{y^{3}_1 - y_1} \right) \cdot y^{2}_1 \right) \cdot  \frac{1}{ y^{n}_{1}}  = \frac{360y_1 - 4}{y^{3}_1 - y_1} \cdot y^{n}_{1} +  \left( 4y_1 - \frac{360y^{2}_1 - 4y_1}{y^{2}_1 - 1}   \right) \cdot  \frac{1}{ y^{n}_{1}} \ \forall n \in \mathbb{N}^{*}$

 

Viết gọn nhất dưới dạng:

$ x_n = \frac{360y_1 - 4}{y^{2}_1 - 1} \cdot y^{n-1}_{1} +  \frac{4y_1 - 360}{y^{2}_1 - 1} \cdot  \frac{1}{ y^{n-2}_{1}} \ \forall n \in \mathbb{N}^{*}$

 

 

Trong đó $ y_1 = 45+2\sqrt{506}$

 

Và bài toán theo đó được giải quyết hoàn toàn.




#741220 Chứng minh rằng $(abc)^2(a^2+b^2+c^2) \leq 3$

Gửi bởi supermember trong 26-08-2023 - 16:54

Sử dụng bất đẳng thức Cauchy cho 4 số:

 

$ 27 (abc)^2 (a^2+b^2+c^2) = (3ab) \cdot (3bc) \cdot (3ca) \cdot (a^2 + b^2 + c^2) \leq \left( \frac{3ab+3bc+3ca + a^2 + b^2 + c^2}{4} \right)^4 $

 

$= \left( \frac{ab+bc+ca + (a + b + c)^2}{4} \right)^4  \leq   \left( \frac{ \frac{(a + b + c)^2}{3} + (a + b + c)^2}{4} \right)^4  =   \left( \frac{(a + b + c)^2}{3} \right)^4 = 3^4 \implies  (abc)^2 (a^2+b^2+c^2) \leq 3$




#741216 $\left\{\begin{matrix} a^{2}_2 +1 = (a_1 +1)(a_3 +1)...

Gửi bởi supermember trong 26-08-2023 - 08:49

Giải hệ phương trình nghiệm nguyên dương:

 

$\left\{\begin{matrix} a^{2}_2 +1 = (a_1 +1)(a_3 +1) & \\ a^{2}_3 +1 = (a_2 +1)(a_4 +1) & \\ a^{2}_4 +1 = (a_3 +1)(a_5 +1) & \end{matrix}\right.$




#740521 Tính giá trị của $ (a+2)(b-10)(c+2)$

Gửi bởi supermember trong 10-07-2023 - 23:40

ĐÃ TIẾP THU Ý KIẾN TỪ THẦY GIÁO PHẠM HẢI ĐĂNG VÀ SỬA LẠI.

Bài này cách nhanh nhất thế này:

 

$ x^2 + x = x(x+1) = y+x = 2a  \implies x(x+1) = 2a $ với $a$ là số nguyên tố.

 

Trường hợp $1$: Nếu $ a | x$ thì $ x = ka$ với $k$ là số nguyên , suy ra: $ x +1 = ka +1  \implies ka(ka +1) = 2a  \implies k(ka+1) = 2$

Suy ra có $4$ trường hợp có thể xảy ra: $ k =1 ; ka+1 = 2$ hoặc  $ k =2 ; ka +1 = 1$ hoặc   $ k = -1 ; ka+1 = -2$ hoặc   $ k = -2 ; ka+1 = -1$

Mà $a$ là số nguyên tố nên dễ thấy chỉ có thể xảy ra: $ k = -1; a =3$

 

Suy ra $ x =  -3; y = 9$

 

Do $ z-y = 2(b-a)$ là số chẵn, nên hiển nhiên $2$ số $ z ; y$ cùng tính chẵn lẻ, suy ra nếu $ \sqrt{z}$ là số nguyên dương thì $ \sqrt{z} -\sqrt{y}=  \sqrt{z} -3$ là số chẵn . Khi đó chỉ có thể xảy ra trường hợp:  $  \sqrt{z} -3 =4$ Suy ra:  $ z = 49$

 

 

$ z+x = 2b \implies 46 = 2b \implies b = 23$

$ y+z = 2c \implies 58 = 2c \implies c = 29$

Thỏa mãn điều kiện $a; \ b; \ c $ là số nguyên tố:

 

Suy ra: $(a+2)(b-10)(c+2) = 5 \cdot 13 \cdot 31 = 2015$

 

Trường hợp $2$: Mà nếu $ a | x+1$ thì  $ x+1 = ka $ với $k$ là số nguyên , suy ra: $ x = ka -1  \implies ka(ka -1) = 2a  \implies k(ka-1) = 2$

 

Suy ra có $4$ trường hợp có thể xảy ra: $ k =1 ; ka-1 = 2$ hoặc    $ k =2 ; ka -1 = 1$ hoặc   $ k = -1 ; ka-1 = -2$ hoặc   $ k = -2 ; ka-1 = -1$

 

Mà $a$ là số nguyên tố nên dễ thấy chỉ có thể xảy ra: $ k =1; a =3$

 

Suy ra $ x = 2; y = 4$

 

Lập luận tương tự ở trên, ta có: $ \sqrt{z} -\sqrt{y}=  \sqrt{z} -2 =4$ Suy ra:  $ z = 36$

 

$ y+z = 2c \implies 38 = 2b \implies c = 20$, loại  trường hợp này vì $c$ phải là số nguyên tố.




#740021 $\left | \frac{a-b}{c} +\frac{b-c}{a}+\frac{c-a}{b}...

Gửi bởi supermember trong 13-06-2023 - 18:47

Không rõ là có cách nào đơn giản để giải bài này không, còn nếu " chém đinh chặt sắt" thì dùng cách dưới đây:

 

 

Cho các số thực dương $a,b,c$ thỏa mãn $\frac{1}{2}\leq a,b,c\leq 1$ . Chứng minh rằng : 
$\left | \frac{a-b}{c} +\frac{b-c}{a}+\frac{c-a}{b}\right |\leq \left ( 1-\frac{\sqrt{2}}{2} \right )^{2}$

 

Ta có: $ \frac{a-b}{c} + \frac{b-c}{a} + \frac{c-a}{b} = \frac{ ab(a-b)+ bc(b-c)+ca(c-a)}{abc}$

 

Đến đây thì ta sử dụng một kỹ năng biến đổi tương đối quen thuộc:

 

$ ab(a-b)+ bc(b-c)+ca(c-a) = ab(a-b) + bc \cdot \left( (b-a) + (a-c) \right) + ca(c-a) $

$= ab(a-b) - bc(a-b) - (c-a)bc + ca(c-a) = (a-b)b(a-c) + (c-a)c(a-b) $

$= - (a-b)b(c-a) + (c-a)c(a-b) = (a-b)(c-a)(c-b) =  - (a-b)(c-a)(b-c) $

 

Suy ra: $ | \frac{a-b}{c} + \frac{b-c}{a} + \frac{c-a}{b} | = \frac{ | (a-b)(c-a)(b-c)|}{abc}$  $(1)$

 

Do vai trò của $a;\ b; \ c$ là như nhau, nên không giảm tổng quát, ta giả sử: $ a \geq b \geq c$

 

Đặt: $ a = b+ x ; \ a = c+ y$, do $ a-c \geq a-b \implies 0 \leq x \leq y \leq \frac{1}{2}$  

 

Đến đây thì ta thu được một dạng gọn, đẹp, không chứa dấu giá trị tuyệt đối:

 

$ \frac{ | (a-b)(c-a)(b-c)|}{abc} = \frac{xy(y-x)}{a(a-x)(a-y)} = g(x;y)$  $(2)$

 

Cố định giá trị của $a; \ y$ sau đó ta sẽ cho $x$ là biến chạy trong khoảng $ [ 0; y]$ rồi khảo sát hàm số bình thường:

 

Xét hàm:  $ f(x) = \frac{ x(y-x)}{a-x}$ ; ta có: $ g(x;y) = k \cdot f(x)$ trong đó $ k = \frac{y}{a(a-y)}$ là hằng số dương.

 

Ta có: $ f^{'} (x) = \frac{ (y-2x)(a-x) + (xy-x^2)}{ (a-x)^2} =  \frac{ x^2 - 2ax+ya}{ (a-x)^2}$

 

Ta đi giải phương trình $ f^{'} (x) = 0$ .

 

Biệt số $ \triangle^{'} = a^2 - ya = a(a-y)$

 

Do đó, theo công thức nghiệm của phương trình bậc $2$, ta thu được $2$ nghiệm: $  x_{ 1; \   2}= a \pm \sqrt{a(a-y)}$ mà do $ x \leq y < a$. Nên phương trình $ f^{'} (x) = 0$ chỉ có thể nhận nghiệm duy nhất $ x_1 = a- \sqrt{a(a-y)} $, dễ thấy $ 0< x_1 < y <  a + \sqrt{a(a-y)} = x_2$

 

Từ dáng điệu của hàm số này, ta thấy: $g(x;y)$  là hàm đơn điệu tăng trên $ [0; x_1]$ và đơn điệu giảm trên $ [x_1 ; y]$

Ta có thể giải thích kỹ hơn đoạn này bằng cách lập luận: $  f^{'} (x) \geq  0$ với $x$ thuộc $ [0; x_1]$  và  $  f^{'} (x) \leq  0$ với $x$ thuộc $ [x_1 ; y]$.

 

Tức là khi $x$ chạy qua $ [ 0; y]$ thì $ g(x;y)$ sẽ đạt giá trị lớn nhất  tại điểm $ x= x_1$ 

 

Suy ra: $ g(x;y) \leq  g(x_1 ;y) =  \frac{y}{a(a-y)} \cdot \left( a+ (a-y)- 2 \sqrt{a(a-y)} \right) = h(a;y) = \frac{y}{a-y} + \frac{y}{a} - \frac{2y}{ \sqrt{a(a-y)}} $  $(3)$

 

Ý tưởng đến đây là tiếp tục " chém đinh chặt sắt", cố định $a$ , coi $ h(a;y)$ là hàm một biến theo biến $y$ rồi khảo sát hàm này trên $ [0; a)$. Đến đây đói rồi, lát làm tiếp.

 

Tiếp tục hành trình:

 

Dễ thấy: $ h(a; y) = y \cdot \left( \frac{1}{ \sqrt{a-y}} - \frac{1}{\sqrt{a}} \right)^2$ 

 

Đến đây bắt đầu tính toán khó rồi, ta chuyển qua dùng ẩn phụ, đặt: $ t = \frac{1}{ \sqrt{a-y}} = \frac{1}{ \sqrt{c}} \leq  \sqrt{2}$, rõ ràng khi $y$ chạy qua $ [0; a)$ thì $t$ chạy qua $ (0; \sqrt{2} ]$ . Và $ y = a - \frac{1}{t^2}$.

 

Xét hàm số $w(t) =  (a - \frac{1}{t^2} ) \left(t-  \frac{1}{\sqrt{a}} \right)^2 = at^2 - 2 \sqrt{a} t + \frac{2}{ t \sqrt{a}} - \frac{1}{at^2}$

 

Suy ra : $w^{'} (t) = 2at - 2 \sqrt{a} - \frac{2}{ \sqrt{a} t^2} + \frac{2}{at^3}$

 

Giờ đơn giản là đi giải phương trình : $w^{'} (t) = 0$, phương trình này tương đương với:

 

$ n(t)= a^2 t^4 - a \sqrt{a} t^3 - \sqrt{a} t +1 =0 \Leftrightarrow  \sqrt{a} t  = 1$

 

$ \Leftrightarrow   t  =  \frac{1}{ \sqrt{a}}  \Leftrightarrow   \frac{1}{ \sqrt{a-y}}   =  \frac{1}{ \sqrt{a}}   \Leftrightarrow  y =0$

 

Từ dáng điệu của hàm $ w(t)$, ta dễ thấy là $w(t)$ đơn điệu tăng trên $ (0; \sqrt{2} ]$.  Lập luận thì cũng gần giống như trên, chú ý là hàm đa thức bậc $4$ với hệ số  bậc $4$ dương sẽ tiến dần đến dương vô cùng khi biến tiến đến vô cùng, là hàm liên tục trên $\mathbb{R}$ mà lại chỉ nhận giá trị  bằng $0$ tại điểm  $t  =  \frac{1}{ \sqrt{a}}$  thì suy ra hàm sẽ nhận giá trị không âm trên toàn bộ đoạn $[0; \sqrt{2} ] \subset \mathbb{R} $  , Suy ra 

 

$ w(t) \leq w( \sqrt{2}) =   \left( a - \frac{1}{2} \right) \cdot \left( \sqrt{2} - \frac{1}{ \sqrt{a}} \right)^2$

 

$ \implies h(a;y) \leq 2a - 2 \sqrt{2} \sqrt{a} - \frac{1}{2a} + \frac{ \sqrt{2}}{ \sqrt{a}} = v(a)$  $(4)$

 

Tới đây là gần xong hết rồi, chỉ còn một chút công đoạn cuối cùng thôi. Cần chắc tay và kiên định với phương pháp đã chọn.

 

Đặt ẩn phụ: $ r =    \sqrt{2a}$ thì $r$ nhận giá trị trong khoảng $ [1; \sqrt{2} ]$

 

$ v(a) = r^2 -2r - \frac{1}{r^2} +   \frac{2}{r} = m(r)$

 

Ta khảo sát hàm $m(r)$ trên  $ [1; \sqrt{2} ]$ , ta có : $m^{'}(r) = 2r -2 + \frac{2}{r^3} -  \frac{2}{r^2}$

 

$ m^{'}(r) = 0 \Leftrightarrow  r-1 + \frac{1}{r^3} - \frac{1}{r^2} =0  \Leftrightarrow  r =1$

 

 

Từ dáng điệu của hàm $ m(r)$, ta dễ thấy là $m(r)$ đơn điệu tăng trên $ [1; \sqrt{2} ]$, Suy ra :

 

$ v(a) = m(r) \leq m( \sqrt{2} ) = \frac{3}{2} - \sqrt{2} = \left( 1 - \frac{1}{\sqrt{2}} \right)^2$ $(5)$

 

Từ $(1)$; $(2)$; $(3)$ ; $(4)$ , $(5)$  suy ra:  $ | \frac{a-b}{c} + \frac{b-c}{a} + \frac{c-a}{b} |  \leq \left( 1 - \frac{1}{\sqrt{2}} \right)^2$

 

Đẳng thức xảy ra khi và chỉ khi $(a;b;c)$ nhận giá trị là bộ số $  \left( 1;  \frac{1}{ \sqrt{2}}; \frac{1}{2} \right)$ hoặc các hoán vị của bộ số này.

 

Bài Toán Theo đó được giải quyết hoàn toàn. Một Bài Toán Rất Đẹp. 

 

Lưu ý: Sau khi xem kỹ lời giải trên thì supermember thấy chỗ này cần hiểu rõ về hàm liên tục để nắm vững lời giải:

 

Ví dụ ở trên: phương trình bậc $4$ : $a^2 t^4 - a \sqrt{a} t^3 - \sqrt{a} t +1 =0$ nếu ta giải trên tập $ \mathbb{R}$ thì chỉ có nghiệm duy nhất $ t = \frac{1}{ \sqrt{a}}$,  ở đây hàm $n(t) = a^2 t^4 - a \sqrt{a} t^3 - \sqrt{a} t +1$ là hàm liên tục trên $ \mathbb{R}$ và tiến dần về $ + \infty $ khi $t$ tiến dần về $  \infty $ nên nếu hàm này nhận giá trị âm tại  $1$ điểm $ t_0$ ( $t_0 \in  \mathbb{R}$ )  nào đó thì theo tính chất hàm liên tục sẽ có ít nhất $2$  giá trị phân biệt $ t_1 ; t_2 $ thỏa $  n(t_1) = n(t_2) = 0$, vô lý. Do ở trên đã chỉ ra phương trình $ n(t) =0$ chỉ có nghiệm duy nhất trên $ \mathbb{R}$.

 

Hoặc, các bạn có thể nhìn thấy hướng giải thích đơn giản hơn,

 

thực chất: $ w^{'} (t) = \frac{2 ( \sqrt{a} t -1)^2 (at^2 +  \sqrt{a} t  +1)}{at^3} \geq 0$

Và: $ m^{'}(r) = \frac{2(r-1)^2 (r^2 +r+1)}{r^3} \geq 0$.




#739052 Tính giá trị của $ (a+2)(b-10)(c+2)$

Gửi bởi supermember trong 05-05-2023 - 18:43

Bài Này Không Hề Mới, Nếu Bạn Nào Đã Biết Lời Giải Thì Có Thể Nhường cho những bạn chưa làm qua được thử sức.

 

Bài Toán:

 

Cho $ x = a+b -c; \ y = a+c - b; \ z = b+c -a$ trong đó $ a ; b ; c $ là những số nguyên tố.

 

Biết rằng: $ x^2 = y $ và $ \sqrt{z} - \sqrt{y}$ là bình phương của một số nguyên tố.

 

Tính giá trị của $ (a+2)(b-10)(c+2)$




#738982 $pq(n+1) = (p+q)(n^2+1)$ với $p, q$ nguyên tố

Gửi bởi supermember trong 02-05-2023 - 12:52

ok, giải bài cho vui hen, do supermember cũng không ngồi máy lâu được nên mỗi ngày làm $1$ tí nhé.

 

Ta xét $2$ trường hợp:

 

Trường Hợp $1$: $ p=q$ thì khi đó, đẳng thức đã cho tương đương với:

 

$ \frac{p}{2} = \frac{ n^2+1}{n+1}$

 

Tương đương với: $ 2n^2 -pn + (2-p) = 0$ $(*)$

 

Để phương trình $(*)$ có nghiệm nguyên thì điều kiện cần là phải có biệt số $ \triangle$ là số chính phương. Tức là: $ \triangle = p^2 - 8(2-p) = p^2 + 8p-16$ là số chính phương.

 

Nhưng mà dễ thấy là $ p^2 \leq  p^2 + 8p-16 < (p+4)^2$ nên chỉ có thể xảy ra các trường hợp :

 

$ p^2 + 8p-16  \in \{ p^2 ; \ (p+1)^2; \ (p+2)^2; \ (p+3)^2 \}$

 

Sử dụng tính chẵn lẻ để so sánh  thì ta dễ thấy ngay $p^2 + 8p-16$  không thể nhận các giá trị $(p+1)^2$ hay $(p+3)^2$

Chẳng hạn, nếu $p^2 + 8p-16 = (p+1)^2$ thì $ 8p-16 = 2p+1$, vô lý, vì $2$ vế trái, phải sẽ khác tính chẵn lẻ.

 

nên chỉ có thể xảy ra $2$ khả năng:

 

Khả năng $1$: Nếu $ p^2 +8p-16 = p^2$ thì $ p=q=2$. Khi thử lại, ta tính trực tiếp được $2$ giá trị $n$ tương ứng là $ 0; \ 1$ đều là những số tự nhiên. Tức là bộ $2$ số  nguyên tố $(p;q)$: $(2; \ 2)$ thỏa mãn điều kiện bài toán. 

 

Khả năng $2$: Nếu $ p^2 +8p-16 = (p+2)^2$ thì $ p=q=5$. Khi thử lại, ta tính trực tiếp được chỉ có $1$ giá trị số tự nhiên  $n$ tương ứng là $ 3$. Tức là bộ $2$ số  nguyên tố $(p;q)$: $(5; \ 5)$ thỏa mãn điều kiện bài toán. 

 

Trường Hợp $2$: $ p \neq q$ thì khi đó, rõ ràng:

 

$ \gcd( pq; \ p+q) =1$ , suy ra : phân số $ \frac{pq}{p+q}$ tối giản  $(1)$

 

$ \gcd( n^2+1;\  n+1) = \gcd( (n-1)(n+1) +2; \   n+1) =  \gcd( 2; \ n+1) \in \{ 1; 2 \} $

 

Khả năng $1$: Nếu  $ \gcd( n^2+1;\  n+1) = 1$ thì rõ ràng phân số: $ \frac{n^2 +1}{n+1}$ là phân số tối giản  $(2)$

 

Từ $(1) ; \ (2)$, suy ra: $\left\{\begin{matrix} pq= n^2 +1 & \\ p+q = n+1 \end{matrix}\right. (**)$

 

Nhưng hệ $(**)$ này vô nghiệm, chứng minh đơn giản bằng phản chứng.

 

Thật vậy, giả sử tồn tại $(p; \ q; \ n)$ thỏa mãn hệ $(**)$ thì do $ (q+p)^2 \geq 4pq \implies (n+1)^2 \geq 4(n^2+1) \implies 2(n^2+1) + (n-1)^2 \leq 0$ , Vô lý !

 

Khả năng $2$: Nếu  $ \gcd( n^2+1;\  n+1) = 2$ thì rõ ràng phân số: $ \frac{ \frac{n^2 +1}{2}}{\frac{n+1}{2}}$ là phân số tối giản  $(3)$

 

Từ $(3) ; \ (1)$, suy ra: $\left\{\begin{matrix} pq= \frac{n^2 +1}{2} & \\ p+q = \frac{n+1}{2} \end{matrix}\right. (***)$

 

Nhưng hệ $(***)$ này cũng vô nghiệm, chứng minh đơn giản bằng phản chứng y như trên:

 

Thật vậy, giả sử tồn tại $(p; \ q; \ n)$ thỏa mãn hệ $(***)$ thì do $ (q+p)^2 \geq 4pq \implies \left( \frac{n+1}{2} \right)^2 \geq 4\cdot \frac{n^2+1}{2} \implies  (n+1)^2 \geq 8(n^2+1) \implies 6(n^2+1) + (n-1)^2 \leq 0$ , Vô lý !

 

Tức là không thể xảy trường hợp $ p \neq q$

 

Như vậy, sau khi xét tất cả các trường hợp có thể xảy ra, ta kết luận chỉ có bộ $2$ số  nguyên tố $(p;q)$ thỏa mãn điều kiện bài toán là: $(2; \ 2); \ (5; \ 5)$ 

 

Bài Toán Theo Đó Được Giải Quyết Hoàn Toàn. :D 




#738979 Tìm Tất cả các cặp số nguyên tố $(p;q)$ : $ \frac{pq...

Gửi bởi supermember trong 02-05-2023 - 10:30

Bài Toán:

 

Tìm Tất cả các cặp số nguyên tố $(p;q)$ thỏa mãn điều kiện:

 

Tồn tại số tự nhiên $m$ để : $ \frac{pq}{p+q} = \frac{m^2 +1}{m+1}$ 




#738921 Hãy chỉ ra một quy trình biến đổi 23 số trên để được số $p$

Gửi bởi supermember trong 30-04-2023 - 10:12

Bài này giải như thế này:

 

Câu $a$:

 

Ta gọi $S_n$ là tổng của các số trên hàng ngang này sau $n$ bước " xóa $2$ số- thay bằng $1$ số "

 

Rõ ràng: $ S_0 = 1 + 2+3+...+ 23 = \frac{23 \times 24}{2} = 23 \times 12$ là số chẳn.

 

Và để ý là cứ sau $1$ bước thì hiệu $S_{n} - S_{n+1} = a+b - |a-b| \in \{ 2a; 2b \}   $ tức là  cứ sau một bước thì các tổng $S_k$ ( $k \in \mathbb{N}$ )này sẽ không thay đổi tính chẳn lẻ.  Mà do $S_0$ là số chẳn, Vậy $S_{22}$ cũng phải là số chẳn.

 

Mà theo giải thiết bài toán thì $S_{22}$ là số nguyên tố, nên chỉ có thể xảy ra trường hợp duy nhất là: $S_{22} = 2$

 

Câu $b$: Sau khi gỉai quyết xong câu $a$ thì chỉ cần để ý như sau:

 

Nếu thực hiện $1$ bước với $2$ số $2 ; 2$ thì ta sẽ bớt đi $2$ số $2$ trong hàng ngang này và có được $1$ số $0$,

 

Nếu thực hiện $1$ bước với $2$ số $2 ; 0$ thì ta sẽ bớt đi $1$ số $0$ trong hàng ngang này và có được $1$ số $2$,

 

nên ý tưởng của ta đơn giản là biến hàng ngang này thành toàn các số $2$, sau đó ta sẽ xoay sở để có số $0$ xuất hiện.

 

Bước $1$: Giờ ta áp dụng $5$ bước với các cặp số $(5;7); \ (9;11); \ (13;15); \ (17;19); \ (21;23)$

 

Ta thu được hàng ngang sau (không quan trọng thứ tự các số):

 

$1; 3;  2; 2; 2; 2; 2 ; 2 ; 4; 6; 8 ; 10; 12; 14; 16; 18; 20; 22$;  tổng cộng là $6$ số $2$ trong hàng ngang này.

 

Bước $2$:  Giờ ta áp dụng $5$ bước với các cặp số $(4;6); \  (8;10); \ (12;14); \ (16;18); \ (20;22)$

 

Ta thu được hàng ngang sau (không quan trọng thứ tự các số):

 

$1; 3; 2; 2; 2; 2 ; 2 ; 2; 2; 2 ; 2 ; 2 $ , tổng cộng là $11$ số $2$  trong hàng ngang này.

 

Bước $3$:  Giờ ta áp dụng $5$ bước với các cặp số $(2;2); \  (2;2); \ (2;2); \ (2;2); \ (2;2)$

 

Ta thu được hàng ngang sau (không quan trọng thứ tự các số):

 

$0; 0; 0; 0; 0; 2; 1 ; 3 $ , tổng cộng là $1$ số $2$ và $5$ số $0$ trong hàng ngang này.

 

 

Bước $4$:

 

Với $7$ bước còn lại thì ta biến đổi như sau:

Lưu ý là nếu chọn $2$ số nào để thực hiện $1$ bước thì ở dưới ta gạch chân $2$ số đó để người đọc dễ theo dõi:

 

$  (\underline{0}; \  \underline{0}; \ 0; \ 0; \ 0; \ 2; \ 1 ; \  3)  \rightarrow  (\underline{0}; \ \underline{0}; \ 0; \ 0; 2; 1 ; 3)  \rightarrow  (\underline{0}; \ \underline{0}; \  0; \ 2; \ 1 ; \  3)$

$  \rightarrow  (\underline{0}; \underline{0}; \ 2; \ 1 ; \  3) \rightarrow  (\underline{0}; \ 2; \  \underline{1} ; \ 3) \rightarrow  (\underline{1}; \ \underline{2} ; \ 3)  \rightarrow  (\underline{1}; \ \underline{3})  \rightarrow 2 $

 

Quá trình xây dựng hoàn tất để có $1$ số $2$ còn tồn tại trên bảng sau đúng $22$ bước, nên bài toán theo đó được giải quyết hoàn toàn.

 

Lưu ý: Với trình độ THCS, nếu chưa được học cách tính tổng: $S_0$ thì có thể chứng minh $S_0$ là số chẵn bằng cách viết:

 

$  S_0 = (2 + 4+ 6+ 8 + 10+ 12+ 14+ 16+ 18+ 20+ 22) + (1+3) + (5+7) + (9+11) + (13+15) + (17+19) + (21+23)$, dễ thấy $S_0$ phải là số chẵn do các tổng con trong mỗi ngoặc đơn đều là số chẵn.




#738829 Xác suất để $2$ số được chọn có tích chia hết cho $6$

Gửi bởi supermember trong 24-04-2023 - 22:18

Lời giải của bạn Chánh là quá đẳng cấp.

 

Lời giải sau là của supermember:

Gọi tập $20$ số nguyên dương đầu tiên là $A$

 

Gọi $ B = \{ 2; 4; 8; 10; 14; 16; 20 \} = \{ 2^1; 2^2; 2^3; 2 \times 5; 2 \times 7; 2^4; 2^2 \times 5 \}$

 

$ C = \{3; 9; 15 \} = \{3^1; 3^2 ; 3^1 \times 5 \}$

 

$ D = \{6; 12; 18 \} = \{6; 6 \times 2; 6 \times 3 \} $

Thì rõ ràng $ | B| = 7;  | C| = 3; |D| = 3$ và  $ | A \backslash (B \cup C \cup D) | = 7$

 

Để đi tìm số các cách chọn để có tích $2$ số chia hết cho $6$, ta đi gián tiếp bằng cách tính các cách chọn để tích $2$ số không chia hết cho $6$.

 

Trường hợp $1$ : Tích $2$ số được chọn không chia hết cho $2$ và cũng không chia hết cho $3$. Trường hợp này thì rõ ràng $2$ số được chọn sẽ thuộc tập hợp  $A \backslash (B \cup C \cup D)$. Số cách chọn trong trường hợp này là $ \binom{7}{2}$

 

Trường hợp $2$ : Tích $2$ số được chọn chia hết cho $2$ và không chia hết cho $3$.

 

Trường hợp này thì rõ ràng $2$ số được chọn đều thuộc tập $B$; hoặc là có $1$ số thuộc tập $B$  và  $1$ số còn lại  sẽ thuộc tập hợp  $A \backslash (B \cup C \cup D)$. Theo quy tắc nhân và quy tắc cộng thì Số cách chọn trong trường hợp này là:  $ \binom{7}{2} + 7 \times 7 = \binom{7}{2} + 7^2$

 

Trường hợp $3$ : Tích $2$ số được chọn chia hết cho $3$ và không chia hết cho $2$.

 

Trường hợp này thì rõ ràng $2$ số được chọn đều thuộc tập $C$; hoặc là có $1$ số thuộc tập $C$  và $1$ số còn lại  sẽ thuộc tập hợp  $A \backslash (B \cup C \cup D)$. Theo quy tắc nhân và quy tắc cộng thì Số cách chọn trong trường hợp này là:  $ \binom{3}{2} + 3 \times 7 = 3 + 3 \times 7$

 

Theo quy tắc cộng thì số cách chọn ra $2$ số phân biệt sao cho tích $2$ số không chia hết cho $6$ sẽ là:

 

$ \binom{7}{2} + \binom{7}{2} + 7^2+ 3 + 3 \times 7 = 6 \times 7+ 49+ 24 = 115$

 

Do đó, xác suất tần tìm bằng: $ \frac{ \binom{20}{2} - 115}{\binom{20}{2}} = \frac{190-115}{190} = \frac{75}{190} = \frac{15}{38}$




#738618 Tìm min $A = \frac{a}{b} + \frac{b...

Gửi bởi supermember trong 15-04-2023 - 09:27

Tìm min $A = \frac{a}{b} + \frac{b}{a}$ với $a \geq 2b$

 

Áp dụng BĐT Cauchy, ta có:

 

$ A = \left( \frac{a}{4b} + \frac{b}{a} \right) + \frac{3a}{4b} \geq 2 \sqrt{ \frac{a}{4b} \cdot  \frac{b}{a}} + \frac{6b}{4b}  = 1 + \frac{3}{2} = \frac{5}{2}$

 

Đẳng thức xảy ra khi và chỉ khi $ a = 2b$




#737965 Tìm các số tự nhiên n sao cho $3^{2^{2n}}+10$ là số nguyên tố.

Gửi bởi supermember trong 23-03-2023 - 18:39

Bài này làm như thế này.

 

Dễ thấy $n = 0$ thỏa mãn yêu cầu bài toán, vì $ 3^{2^{0}} + 10 = 3^1 + 10 = 13$ là số nguyên tố.

 

Ta sẽ chứng minh bằng quy nạp rằng với mọi số nguyên dương $n$ thì $ 3^{2^{2n}} + 10 = 3^{4^{n}} + 10$ chia hết cho $13$ , và do số này hiển nhiên lớn hơn $13$ nên nó không thể là số nguyên tố.

 

Thật vậy, với $n =1$ thì $ 3^{2^{2}} + 10 =  3^4 + 10 = 91 = 7 \cdot 13 $ , chia hết cho $13$.

 

Vậy khẳng định đúng với $n=1$.

 

Giả sự khẳng định đúng đến $n$, tức là : $ 3^{4^{n}} + 10$ chia hết cho $13$ $(*)$ thì khi đó:

 

 $ 3^{2^{2(n+1)}} + 10 = 3^{4^{n+1}} + 10 =  (3^{4^{n+1}} - 3^{4^{n}}) + (3^{4^{n}}  + 10)  \ (**)  $

 

Mà: $(3^{4^{n+1}} - 3^{4^{n}}) = 3^{4^{n}} \cdot \left( 3^{3\cdot 4^{n}} - 1 \right) =  3^{4^{n}} \cdot \left( (3^{3})^{4^{n}} - 1 \right) = 3^{4^{n}} \cdot \left( 27^{4^{n}} - 1 \right)$

 

$27 \equiv 1  \pmod {13}$ $ \implies 27^{4^{n}} \equiv 1 \pmod{13}  \implies  13 | 27^{4^{n}} - 1$

 

Suy ra: $(3^{4^{n+1}} - 3^{4^{n}}) $ chia hết cho $13$ $(***)$

 

Từ $(*)$; $(**)$; $(***)$ suy ra khẳng định  đúng đến $n+1$ và do đó, theo nguyên lý quy nạp Toán Học thì khẳng định được chứng minh hoàn toàn.

 

Dẫn đến kết luận: Chỉ có duy nhất một số tự nhiên thõa mãn yêu cầu bài toán là $n=0$.




#737897 Chứng minh rằng với mọi số nguyên dương $n,$ số $2^{3^...

Gửi bởi supermember trong 21-03-2023 - 12:00

Bài này giải như thế này.

 

Bước 1: Ta đi chứng minh mọi ước nguyên tố của số $2^{3^{n}}+1$ đều phải có dạng $8k+3$ hoặc $8k+1$.

 

Thật vậy, giả sử : $ 2^{3^{n}}+1 \equiv 0 \pmod p \implies  2^{3^{n}} \equiv -1  \pmod p $

 

$ \implies  2^{3^{n} +1} \equiv -2  \pmod p  \implies  \left(2^{\frac{3^{n} +1}{2}} \right)^2 \equiv -2  \pmod p $

 

Tức là $-2$ là số chính phương $ \pmod p$.

 

$ \implies \big( \frac{-2}{p} \big) =   \big( \frac{-1}{p} \big) \cdot \big( \frac{2}{p} \big) =1$

 

Nên chỉ có thể xảy ra $2$ trường hợp:

 

Trường hợp $1$:  $\big( \frac{-1}{p} \big) = \big( \frac{2}{p} \big) =1$ 

Mà : $ \big( \frac{-1}{p} \big) = (-1)^{\frac{p-1}{2}} ;  \big( \frac{2}{p} \big) = (-1)^{\frac{p^2-1}{8}}$ 

Tức là : $p$ phải có dạng $8k+1$ 

 

Trường hợp $2$:  $\big( \frac{-1}{p} \big) = \big( \frac{2}{p} \big) = -1$ 

Mà : $ \big( \frac{-1}{p} \big) = (-1)^{\frac{p-1}{2}} ;  \big( \frac{2}{p} \big) = (-1)^{\frac{p^2-1}{8}}$ 

Tức là :  $p$ phải có dạng $8k+3$ 

 

Mà rõ ràng  $2^{3^{n}}+1$  cũng có dạng $8k+1$, suy ra khi viết $2^{3^{n}}+1$ dưới dạng tích của các ước số nguyên tố (không nhất thiết phân biệt), thì phải có đúng một số chẵn các ước nguyên tố dạng $8k+3$. Các ước nguyên tố lẻ còn lại (nếu có) thì có dạng $8k+1$. Ta gọi tính chất này là tính chất $ \alpha$.

 

Bước $2$: Ta đi chứng minh số $  (2^{3^{n}})^2 -     2^{3^{n}}+1$ cũng có tính chất $ \alpha$ .

 

Thật vậy, rõ ràng: $  2^{3^{n+1}}+1 = (2^{3^{n}}+1) \cdot ( (2^{3^{n}})^2 -     2^{3^{n}}+1)$ mà  $  2^{3^{n+1}}+1 ; \ 2^{3^{n}}+1 $ đều có tính chất  $ \alpha$. Nên hiển nhiên $  (2^{3^{n}})^2 -     2^{3^{n}}+1$ cũng có tính chất $ \alpha$ .

 

Bước $3$: Ta đi chứng minh $  (2^{3^{n}})^2 -     2^{3^{n}}+1$ phải có ít nhất $1$ ước nguyên tố $p$ dạng $8k+3$ mà $ p$ không phải ước nguyên tố của $2^{3^{n}}+1$ $(*)$.

 

Chứng minh điều này không khó:

 

$   \left( 2^{3^{n}}+1 \right)^2 -  \big( (2^{3^{n}})^2 -     2^{3^{n}}+1 \big) = 3 \cdot 2^{3^{n}}  \ \ (**)$ 

 

Ta để ý điều sau: $ 3 $ $  ||   (2^{3^{n}})^2 -     2^{3^{n}}+1 $ , chứng minh đơn giản bằng cách viết: $ 2^3 = 3^2 -1$.

 

Nên để $ (2^{3^{n}})^2 -     2^{3^{n}}+1$ có tính chất $ \alpha$ thì  $ (2^{3^{n}})^2 -     2^{3^{n}}+1$  phải có ít nhất một ước nguyên tố lẻ $p$ khác $3$ có dạng $ p = 8k+3$, nếu $p$ cũng là ước số của số  $2^{3^{n}}+1$ thì từ  $(**)$ suy ra $ p |  3 \cdot 2^{3^{n}} $ , vô lý.

 

Do đó $(*)$ được chứng minh.

 

Bước $4$: Chứng minh khẳng định bài toán bằng quy nạp.

 

Với $n=1$, $2$ ta dễ thấy khẳng định bài toán đúng do:

 

 Số $2^{3^{1}}+1 = 3^2$ có đúng $1$ ước nguyên tố dạng $8k+3$ là $3$

 

Số $2^{3^{2}}+1 = 3^3 \cdot 19$ có đúng $2$ ước nguyên tố dạng $8k+3$ là $3 ; \ 19$

 

Giả sử khẳng định đúng đến $n$, $ n \geq 2$ thì:

 

 $  2^{3^{n+1}}+1 = (2^{3^{n}}+1) \cdot ( (2^{3^{n}})^2 -     2^{3^{n}}+1)$ , trong đó  $  2^{3^{n}}+1$ có ít nhất $n$ ước nguyên tố phân biệt dạng $8k+3$ (Giả thiết quy nạp). Còn $ (2^{3^{n}})^2 -     2^{3^{n}}+1$ có ít nhất $1$ ước nguyên tố dạng $8k+3$, số nguyên tố này không phải ước của $  2^{3^{n}}+1$ (theo  $(*)$ ) .

 

Suy ra $  2^{3^{n+1}}+1$ phải có ít nhất $n+1$ ước nguyên tố phân biệt dạng $8k+3$.

 

Do đó, khẳng định đúng đến $n+1$, và theo nguyên lý quy nạp toán học, khẳng định đúng với mọi số nguyên dương $n$.

 

Ta thấy rằng bài toán theo đó được giải quyết hoàn toàn.